Conjecture liée à la loi Kolmogorov 0-1 (pour les événements)


8

Laisser (Ω,F,P)être un espace de probabilité. Conjecture:

Supposons que nous ayons les événements st , ou . Il existe une séquence indépendante d'événements stA1,A2,... Anσ(An,An+1,...)P(A)=01B1,B2,...

τAn:=nσ(An,An+1,...)=nσ(Bn,Bn+1,...):=τBn

Est-ce vrai?


Je pense qu'il existe une fonction st sont indépendantes pour que nous puissions choisir . Est-ce vrai? Pourquoi pourquoi pas? Sinon, comment puis-je prouver ou infirmer la conjecture ci-dessus? Si c'est vrai, je pense que cela peut être prouvé en modifiant la preuve de la loi Kolmogorov 0-1 (pour les événements).f:NNAf(n)Bn=Af(n)


Peut-être que l'une de ces sous-séquences d'ensembles est indépendante:

An

A2n,A2n+1

A3n,A3n+1,A3n+2

Amn,Amn+1,Amn+2,...,Amn+(m1)

Je pense que nous avons ça

τAn=τAmn+i:=nσ(Amn+i,Am(n+1)+i,...)

où et .mNi{0,1,2,...,m1}


Il semble que nous ayons besoin d'un tel , s'il existe, pour satisfaire la condition suivante:f(n)

(**)σ(Af(n),Af(n+1)...)σ(An,An+1,...)

ce qui, je suppose, est vrai si (et seulement si?)f(n)n .


Autres candidats possibles pour :f(n) (supposons que les variables sont st est satisfait. Si besoin est, ou aussi.)f:NN()f(n)n

  1. i=0maini

  2. 2n,3n,...

  3. i=1mbicin

  4. tn,tn ( je suppose quet>e1/e )

  5. i=1mbicin,i=1mbicin

  6. linear combination of trigonometric functions,linear combination of trigonometric functions

  7. Some linear combination of the above,Some linear combination of the above


En supposant que la conjecture est vraie , je suppose qu'il n'est pas nécessaire de trouver qui fonctionne pour toutes les séquences possibles d'événements car un tel peut même ne pas exister.f(n)A1,A2,...f(n)


Pour réfuter la conjecture : je suppose que nous devons montrer qu'une telle séquence étant indépendante implique que tail ne sera jamais égal à tail puisque tail sera trivial par Kolmogorov 0-1 Law (pour les événements).BnBnAnBnP

Quelque chose qui pourrait aider: nous pourrions montrer que ou et n'est pas indépendant, mais je ne suis pas sûr que la conjecture soit réfutée parce que nous pourrait construire des qui ressemblent à: Anσ(Af(n),Af(n+1),...),P(A)=01nN,Af(n),Af(n+1),...Bn

  1. Bn=An+1An
  2. Bn=AnAn1,A0=
  3. Bn=mAmn
  4. Bn=mAmn
  5. B2n=mAmn,B2n+1=mAmn
  6. Bn=lim supmAmn
  7. Bn=lim infmAmn
  8. B2n=lim supmAmn,B2n+1=lim infmAmn

Cela ne veut pas dire, bien sûr, que ces satisfont mais que n'a pas besoin d'être sous la forme .BnτAn=τBnBnAf(n)


Borel-Cantelli:

  1. Si . Par conséquent, est indépendant.nP(An)<0=P(lim supAn)=P(lim supAmn) mNBm=lim supAmn

  2. Si , alors peut - être cette extension de Borel-Cantelli ? Je ne suis pas sûr de bien comprendre ou comment cela serait utile. Je ne pense pas que nous puissions conclure quoi que ce soit si nous avons .nP(An)=P(lim supAn)

  3. Ensuite, il y a le cas de mais les conditions précédentes ne sont pas remplies.nP(An)=


1
Peut-être une preuve par construction, où ? B1=A1,B2=A2A1,
jbowman

1
Pour moi, cette conjecture ne semble pas être vraie à moins que vous ajoutiez des conditions supplémentaires ou que vous vouliez dire que les deux achèvements de l'algèbre d'accord (ce qui est presque trivial). Cependant, je ne vois pas de contre-exemple. σ
P.Windridge

1
Dans tous les cas, je pense que vous pouvez commencer par la question (plus simple): "Soit un espace de probabilité. Supposons est une -algèbre générée numériquement et qui ou pour tout événement . Existe-t-il une séquence d'événements indépendants in avec queue -algèbre ?(Ω,F,P)GFσP(A)=01AGB1,B2,FσG
P.Windridge

2
Une -algèbre est générée numériquement s'il existe st . Il est simple de trouver des exemples où l'algorithme tail n'est pas généré de façon dénombrable. σGF1,F2,G=σ(F1,F2,)σ
P.Windridge

2
Plus généralement, une sous- algèbre d'une algèbre générée numériquement peut ne pas être elle-même générée numériquement! Regardez en fait l'exercice 1.1.18 dans math.mit.edu/~dws/175/prob01.pdfσσ
P.Windridge

Réponses:


7

Si vous voulez que les événements soient indépendants de manière intéressante (pas simplement parce que ou ) alors la conjecture est fausse.BnP(Bn)=0P(Bn)=1

Voici un exemple pédant. Supposons que est un espace de probabilité suffisamment riche. (Ω,F,P)

Soit être -null, c'est-à-dire . Prenez , de sorte que la queue -algèbre soit .AFPP(A)=0Ai=AσG={,A,Ac,Ω}

Notez qu'en particulier, est fini.G

Supposons maintenant que soit une séquence d'événements indépendante avec délimitée par et . Ensuite, la queue -algèbre n'est pas générée de façon dénombrable. (Voir par exemple l'exercice 1.1.18 http://math.mit.edu/~dws/175/prob01.pdf , qui utilise un argument comme je l'ai expliqué ci-dessus - tout -trivial -algebra généré un atome de masse , mais n'a pas un tel atome).B1,B2,P(Bn)01σHPσ1H

Donc, est fini mais n'est même pas généré de façon dénombrable.GH


Édition 2: si vous acceptezP(Bn)=0 vous pouvez répliquer tout -trivial -algebra généré numériquement . Plus en détail, supposons que est généré par les événements . Si est -trivial alors les sont tous indépendants, en vertu d'être nuls (ou étant nuls). Faites maintenant une construction triangulaire pour les événements : , , . PσGE1,E2,GFGPEnEncBB1,1=E1B2,1=E1,B2,2=E2,,Bk,j=Ej1jk

Alors est une séquence dénombrable (avec un ordre naturel pour les indices) d'événements indépendants dont la queue -algèbre est .(Bk,j)σG

Donc, ici, je pense que c'est la question clé: supposons que est une queue mathématique générée numériquement -algèbre (provenant d'événements non nuls qui pourraient être dépendants). Peut être réalisé comme la queue -algèbre pour certains événements nuls?GPσGσ

Édition 1: une zone grise se produit si vous acceptez , bien que cela ne semble pas être l'idée maîtresse de la question d'origine.P(Bn)0


Merci P.Windridge, mais je ne suis pas sûr de comprendre. 1 Si nous incluons ou , la conjecture est (trivialement?) Vraie? 2 Est-ce ce que vous essayez de prouver dans Edit 2? Si oui, votre égal à mon ? J'ai édité OP pour sténographieP(Bn)=01GτAn
BCLC

J'ai lu l'exercice. ? H=τBn
BCLC

Salut BCLC, (1) Je dis que si nous incluons P(Bn)=0 alors la conjecture est vraie pour tous les choix des événements A1,A2, qui ont une "belle" queue σ-algèbre (où "agréable" signifie ici généré numériquement). (2) OuiG=τ(An) et H est ton τ(Bn). NB l'exercice lié utilise "An"pour ce que devrait être votre Bn, et "Bn"dénote une séquence candidate de génération d'événements (utilisée pour obtenir une contradiction.
P.Windridge

Je ne suis pas sûr de suivre. Juste à partir des hypothèses, le AiSONT indépendants?
BCLC

Dans l'exercice 1.1.18 de math.mit.edu/~dws/175/prob01.pdf , leAisont des événements indépendants, que vous devriez considérer comme Biest dans votre conjecture. C'est bien ce que vous demandiez?
P.Windridge
En utilisant notre site, vous reconnaissez avoir lu et compris notre politique liée aux cookies et notre politique de confidentialité.
Licensed under cc by-sa 3.0 with attribution required.